Translate

Wednesday, July 9, 2014

Bài toán:(Đề nghi HSG 10 ĐBDHBB) Tìm các nguyên dương $x_1<x_2<...<x_m$ ($m\in N^*$) thỏa mãn:

                                              $x_1^2+x_2^2+...+x_m^2\le \dfrac{2m+1}{3}(x_1+x_2+...+x_m)$
Lời giải:
 Trước hết, ta cần chứng minh rằng, với $x_1<x_2<...<x_m$. Ta có:
                                  $x_1^2+x_2^2+...+x_m^2\ge \dfrac{2m+1}{3}(x_1+x_2+...+x_m)$
   Thật vậy, với $m=1$, đúng
   Giả sử khẳng định trên đúng tới $m=k\ge 1$, tức là:
                                    $x_1^2+x_2^2+...+x_k^2\ge \dfrac{2k+1}{3}(x_1+x_2+...+x_k)$
   Với $m=k+1$, ta cần chỉ ra:
                                    $\sum_{i=1}^{k+1}x_i^2\ge \dfrac{2k+1}{3}\sum_{i=1}^kx_i+\dfrac{2k+1}{3}x_{k+1}+\dfrac{2}{3}\sum_{i=1}^{k+1}x_i$
   Sử dụng giả thiết qui nạp, ta cần chứng minh:
                                    $a_{k+1}^2\ge \dfrac{2}{3}\sum_{i=1}^{k}x_i+\dfrac{2k+3}{3}x_{k+1}$
   Để ý rằng, 
                                    $\sum_{i=1}^kx_i\le (x_{k+1}-1)x_{k+1}$
                               $\Leftrightarrow \sum_{i=1}^kx_i +\dfrac{2k+3}{3}x_{k+1}\le \dfrac{x_{k+1}^2-x_{k+1}+(2k+3)x_{k+1}}{3}$
                               $\Leftrightarrow x_{k+1}\ge k+1$
 Điều này luôn đúng vì $x_i$ nguyên dương phân biệt. Do đó khẳng định được chứng minh.
 Trở lại bài toán, dễ dàng thấy ngay rằng,  
                                       $x_1^2+x_2^2+...+x_m^2\le \dfrac{2m+1}{3}(x_1+x_2+...+x_m)$
 xảy ra khi và chỉ khi    $x_1^2+x_2^2+...+x_m^2= \dfrac{2m+1}{3}(x_1+x_2+...+x_m)$, tức là $x_1=1, x_2=2, x_3=3,..., x_n=n$

No comments: